1 / 11

Problem 1

Problem 1. Consider the populations that have the genotypes shown in the following table: A. Which of the populations are in Hardy-Weinberg equilibrium B. What are the p and q in each population?

Download Presentation

Problem 1

An Image/Link below is provided (as is) to download presentation Download Policy: Content on the Website is provided to you AS IS for your information and personal use and may not be sold / licensed / shared on other websites without getting consent from its author. Content is provided to you AS IS for your information and personal use only. Download presentation by click this link. While downloading, if for some reason you are not able to download a presentation, the publisher may have deleted the file from their server. During download, if you can't get a presentation, the file might be deleted by the publisher.

E N D

Presentation Transcript


  1. Problem 1 • Consider the populations that have the genotypes shown in the following table: • A. Which of the populations are in Hardy-Weinberg equilibrium • B. What are the p and q in each population? • In population 5, it is discovered that the A to a mutation rate is 5x10-6 and that the reverse mutation is negligible. What must be the fitness of the a/a pehnotype? (q2=u/s where u is the mutation rate and s is the coefficient of selection)

  2. Answer Problem 1 A. Which of the populations are in Hardy-Weinberg equilibrium • B. What are the p and q in each population? • For each the p and q must first be calculated • #1 p=1+1/2(0)=1 q=1-1=0 • #2 p=0+1/2(1)=.5 q=1-.5=.5 • #3 q=1+1/2(0)=1 p=1-1=0 • #4 p=0.5+1/2(0.25)=0.625 q=1-0.625=0.375 • #5 p=0.986049+1/2(0.013902)=0.993 q=1-0.993=0.007 • From the p and q simply calculate A/A= p2, A/a=2pq, a/a=q2 • #1 A/A= p2=1x1=1 so yes • #2 A/a=2pq=2x.5x.5=0.5 so no • #3 a/a=q2=1x1=1 so yes • #4 A/A=p2=0.625x0.625=0.391 so no • #5 A/A=p2=0.993x0.993=0.986 A/a=2pq=2x0.993x0.007=0.014 a/a=q2=0.007x0.007=0.000049 so yes

  3. Problem 1 • In population 5, it is discovered that the A to a mutation rate is 5x10-6 and that the reverse mutation is negligible. What must be the fitness of the a/a phenotype? (q2=u/s where u is the mutation rate and s is the coefficient of selection) • we know that the fitness is related to the coefficient of selection by the equation f=1-s. So s=1-f. Therefore 0.000049=5x10-6/s s=0.102 and f=0.898 • So the fitness of the a/a phenotype is 0.898

  4. I II III Problem 2 • A woman (II2 in the pedigree) wishes to know the probability that she is a carrier of Duchenne muscular dystrophy. • a. What is the probability if she has another affected male child • b. What is the probability if she has another unaffected male child • c. What is the probability if she has another unaffected female child • d. If she has another unaffected male child what is the probability she will have another affected child

  5. I II III Answer Problem 2 • A woman (II2 in the pedigree) wishes to know the probability that she is a carrier of Duchenne muscular dystrophy. This problem is best solved using Bayes’ Theorem It is easiest to make a table Ancestral information Considers children So the probability she is a carrier is 1/9

  6. Answer Problem 2 a. What is the probability if she has another affected male child If she has an affected child then she is a carrier =1 I II III

  7. I II III Answer Problem 2 • b. What is the probability if she has another unaffected male child This problem is best solved using Bayes’ Theorem It is easiest to make a table If she has another unaffected male child then the conditional probability will change So the probability she is a carrier is 1/17

  8. Answer Problem 2 • c. What is the probability if she has another unaffected female child This problem is best solved using Bayes’ Theorem It is easiest to make a table I If she has a female child no additional information is gained because the disease is X-linked recessive II III So the probability she is a carrier is 1/9

  9. Answer Problem 2 • d. If she has another unaffected male child what is the probability she will have another affected child This problem is best solved using Bayes’ Theorem It is easiest to make a table I If she has another unaffected male child then the conditional probability will change II III So the probability she is a carrier is 1/17 if she had four unaffected boys. To pass on the recessive allele is ½ and the chance the child is male is 1/2. So the chance she has an affected child is 1/17X1/2X1/2=1/68

  10. Problem 3 • What is the probability that the a healthy member of the general population is a carrier of cystic fibrosis if she tests negative on the common mutation screening analysis (DF508 mutation). It is known that the incidence of cystic fibrosis in the general population is 1 in 1600 and the common mutation test detects 75 per cent of all cycstic fibrosis alleles.

  11. Answer Problem 3 • First you should make a table • The prior probability. • Is a carrier • We know that the incidence of cystic fibrosis in this case is 1/1600. Because cystic fibrosis is an autosomal recessive disease a/a=q2=1/1600 so q=.025 and p=0.975 (p+q=1). The possibility of being heterozygous is 2pq=2x.025x0.975=0.049 • Is not a carrier • 1-0.049=0.951 • Conditional probability • Is a carrier • fequals chance she does not have the mutation detected by the test which 25% • Is not a carrier • equal 1 • Joint probability • just priorxconditional • Is a carrier • 0.25x0.049=0.012 • is not carrier • 1x0.951 • Posterior • Is not carrier • 0.951/(0.012+0.951)=0.987 • Is a carrier • 0.012/(0.012+0.951)=0.012

More Related